2019 MPFG Problem 15: Difference between revisions
Created page with "==Problem== How many ordered pairs <math>(x, y)</math> of real numbers <math>x</math> and <math>y</math> are there such that <math>-100 \pi \leq x \leq 100 \pi</math>, <math>-..." |
|||
| Line 3: | Line 3: | ||
==Solution 1== | ==Solution 1== | ||
According to the < | According to the <imath>\tan</imath> angle sum trigonometric identity, | ||
<cmath> | <cmath> | ||
| Line 17: | Line 17: | ||
</cmath> | </cmath> | ||
The two equations < | The two equations <imath>\tan x \cdot \tan y = \frac{20.19}{\tan 20.19} - 1</imath> and <imath>\tan x + \tan y = 20.19</imath> create a set of [[Vieta's Formulas|Vieta's formulas]] for | ||
<cmath> | <cmath> | ||
| Line 23: | Line 23: | ||
</cmath> | </cmath> | ||
whose discriminant < | whose discriminant <imath>\Delta</imath> is obviously greater than 0. This indicates that there must be a constant value for the set <imath>(\tan x, \tan y)</imath>. | ||
Assume that < | Assume that <imath>\tan x > \tan y</imath>. <imath>\tan x</imath> is represented by the upper blue line, <imath>\tan y</imath> is represented by the lower red line. | ||
[[File:Forgot_line.png|710px|center]] | [[File:Forgot_line.png|710px|center]] | ||
As we can see, each value of < | As we can see, each value of <imath>x</imath> matches a value of <imath>y</imath> on the other side of the <imath>y</imath>-axis. Because <imath>x + y = 20.19</imath>, which is approximately <imath>6.42 \pi</imath>, 6 values of <imath>x/y</imath> close to <imath>-100 \pi</imath> cannot be taken. | ||
There are < | There are <imath>200 - 6 = 194</imath> values of <imath>(x, y)</imath> when <imath>\tan x > \tan y</imath>. Doubling this number, we get <imath>\boxed{388}</imath>. | ||
~cassphe | ~cassphe | ||
~edited by aoum | ~edited by aoum | ||
Latest revision as of 08:58, 7 November 2025
Problem
How many ordered pairs
of real numbers
and
are there such that
,
,
, and
?
Solution 1
According to the
angle sum trigonometric identity,
The two equations
and
create a set of Vieta's formulas for
whose discriminant
is obviously greater than 0. This indicates that there must be a constant value for the set
.
Assume that
.
is represented by the upper blue line,
is represented by the lower red line.

As we can see, each value of
matches a value of
on the other side of the
-axis. Because
, which is approximately
, 6 values of
close to
cannot be taken.
There are
values of
when
. Doubling this number, we get
.
~cassphe
~edited by aoum